Тёмный

the most fun derivative of x^x^x^... 

blackpenredpen
Подписаться 1,3 млн
Просмотров 349 тыс.
50% 1

Derivative of infinite power tower x^x^x^...
Can you solve x^x^x^...=2 vs. x^x^x^...=3 • Video
Subscribe to ‪@blackpenredpen‬ for more fun math videos
#calculus #blackpenredpen #math

Опубликовано:

 

15 сен 2024

Поделиться:

Ссылка:

Скачать:

Готовим ссылку...

Добавить в:

Мой плейлист
Посмотреть позже
Комментарии : 659   
@blackpenredpen
@blackpenredpen 11 месяцев назад
Can you solve x^x^x^...=2 vs. x^x^x^...=3 ru-vid.com/video/%D0%B2%D0%B8%D0%B4%D0%B5%D0%BE-WAjgupg3hDk.html
@ethannguyen2754
@ethannguyen2754 5 лет назад
Girl : So how many exes do you have? Me : Well, I have an infinite amount of Xs
@KshitijKale
@KshitijKale 4 года назад
Meme patch update 2020 Me: Yes
@Qermaq
@Qermaq 4 года назад
I don't want to find my x, and don't ask me about y, either.
@bartlomiejodachowski
@bartlomiejodachowski 3 года назад
Hromosomes
@MisterPenguin42
@MisterPenguin42 3 года назад
Don't worry, I can differentiate them all.
@Vivek-io3gj
@Vivek-io3gj 2 года назад
You two must integrate well
@py8554
@py8554 3 года назад
Did a bit of research and now I know that the function x^x^x^x^..... (which can also be viewed as the solution for y in the equation x^y=y or x=y^(1/y)) is called "tetration of infinite height" and this function converges for e^(-e) < x < e^(1/e), roughly the interval from 0.066 to 1.44. Also this function is related to the Lambert W-function. In fact y = WC-In x)/ (In x) where Wis the Lambert W-function, which is defined as the functional inverse of xe^x with respect to x.
@MiroslavOstapenko
@MiroslavOstapenko Год назад
Knew the first part, but the relation with the W function blew my mind🤯❤️
@tangobravo5752
@tangobravo5752 Год назад
Wis is the mentor to Beerus in Dragonball Super
@46pi26
@46pi26 6 лет назад
Instructions unclear; impressed too many girls
@redrestt
@redrestt 5 лет назад
Instructions were clear, mate
@lyrimetacurl0
@lyrimetacurl0 5 лет назад
Instructions clear: mate
@Prasen1729
@Prasen1729 4 года назад
How stupid you are. Lol
@megauser8512
@megauser8512 4 года назад
lol
@ashtonsmith1730
@ashtonsmith1730 4 года назад
How many?
@GustavoOwned
@GustavoOwned 6 лет назад
4:52 You was smiling and that all. But all of sudden you became serious and said "Blue." Lmao haha
@samp-w7439
@samp-w7439 3 года назад
Was looking for this comment
@samp-w7439
@samp-w7439 3 года назад
":) I will do this in..." ":| Blue. Of course."
@johnleandrepermison7175
@johnleandrepermison7175 4 месяца назад
Bprpbp of course 😂😂
@soupe2000
@soupe2000 5 лет назад
In the rendez-vous : girl : "So, what can you do?" me : "I can differentiate x^x^x^x^... " *she falls in your arms*
@legendarysom5605
@legendarysom5605 4 года назад
I went and she slapped me and said not maths
@RoyEduworks
@RoyEduworks 4 года назад
ru-vid.com/video/%D0%B2%D0%B8%D0%B4%D0%B5%D0%BE-E29xoux3i_Y.html
@findingdory3040
@findingdory3040 4 года назад
I doubt it
@MatthewsPersonal
@MatthewsPersonal 3 года назад
This works, can confirm. It took 12 hours of saying x^x though
@Theraot
@Theraot 7 лет назад
3:37 now that is what I cal huge integration
@blackpenredpen
@blackpenredpen 7 лет назад
Alfonso J. Ramos hahhahhahaha!!!!!
@GustavoOwned
@GustavoOwned 6 лет назад
LMAO
@clyde__cruz1
@clyde__cruz1 6 лет назад
Integration?
@simohayha6031
@simohayha6031 5 лет назад
@@clyde__cruz1 that arrow he draws is like a huge integration symbol
@MaggotNr9
@MaggotNr9 5 лет назад
I had the same initial wtf thought xD
@AndDiracisHisProphet
@AndDiracisHisProphet 7 лет назад
BPRP does know the right kind of girls^^
@blackpenredpen
@blackpenredpen 6 лет назад
Hahahaha, thanks!!!!
@BeauBreedlove
@BeauBreedlove 5 лет назад
And the rare quadruple factorial
@manofmystery5191
@manofmystery5191 4 года назад
You’re profile picture is Paul Dirac. You are now one of my favorite people on the internet.
@mohammedayankhan4497
@mohammedayankhan4497 4 года назад
@@blackpenredpen I have differentiated x[4]x with respect to x, where this a[b]c represents infix notation.
@brooksgunn5235
@brooksgunn5235 7 лет назад
Freshman boys, if you want to have the girls rolling in, show them the full solution. I cannot stress this enough. 👏
@blackpenredpen
@blackpenredpen 7 лет назад
lol!!!
@bloodyadaku
@bloodyadaku 6 лет назад
You can technically reduce that even more, as ylnx is equal to lny, so in the denominator you'd simply have x-xln(x^x^x^...)
@manuelferrer6501
@manuelferrer6501 2 года назад
Same though, but x ln(x^x^...) could also just be = ln( x^x^x...) ?
@F_A_F123
@F_A_F123 8 месяцев назад
​@@manuelferrer6501 no, x * ln(x^x^ . . .) = ln((x^x^ . . .)^x) (1), which is not ln(x^x^ . . .) (2) (e. g. if we substitute √2 for x in (1) we'll get ln(2^√2) ≈ 0.980, and if we substitute that in (2) we'll get ln(2) ≈ 0.693.
@DoctorT144
@DoctorT144 4 года назад
The infinite power tower converges on: { e^(-e) < x < e^(1/e) }, approx. { 0.066 ~< x ~< 1.445 } Also fun fact, the square root of 2 raised to its own power infinitely many times is actually equal to 2. Try it in a calculator!
@paulchapman8023
@paulchapman8023 4 года назад
Maybe I made an error, but my efforts to evaluate sqrt(2)^sqrt(2)^sqrt(2)^... aren’t giving me a clear answer. Here’s what I did: Let x = sqrt(2)^(sqrt(2)^(sqrt(2)^...)) x = sqrt(2)^x x = 2^(x/2) Squaring both sides gives: x^2 = 2^x Which has two positive solutions: x = 2 or x = 4 I know that there is another point where the functions x^2 and 2^x intersect, where x < 0, but since sqrt(2)^sqrt(2)... > 0, I disregarded it. I assume there is a similar process of elimination that rules out x = 4 as a solution, but what is it?
@GhostHawk272
@GhostHawk272 2 года назад
@@paulchapman8023 when you squared, you created an extra solution
@morgoth4486
@morgoth4486 Год назад
​@@GhostHawk272he didnt, 4 satisfies second equation (before squaring) as well
@Riolupai
@Riolupai Год назад
What about 3rd root of 3, 4th root of 4, etc (or alternatively sqrt(3), sqrt(4), etc)
@MouhibBayounes
@MouhibBayounes 5 месяцев назад
​@@Riolupai the third root of 3 gets you two solutions: The true solution, 2.478 The second solution that you have to rule out, 3 The fourth root of 4 is sqrt(2) 😂 The fifth root of 5, is 1.7649 (true solution) Or 5 (other solution) I'm refering by the "true solution", to the result of a^a^a^... But if you take for example 5=x^x^x^... Then you will find x=5^1/5 So then 5 would be either "the true solution" or "the other solution" to 5^1/5 (And we found out it is the "other solution")
@someonelol3404
@someonelol3404 3 года назад
Interesting! I tried on Georgebra to add more and more Xs in the denominator, but I observed that the value of f(0) was oscillating between 1 and 0 when I add more Xs, so it is not converging. But if you plug 0 in your incredible derivative, we see that it gives a division by 0, which correlates to the impossible value of f(0)! Awesome mate!
@esajpsasipes2822
@esajpsasipes2822 Год назад
It oscillates because 0^0 = 1 and 0^1 = 1. So at the top-most x^x, it's 0^0 = 1, then 0^1 = 0, and it repeats until you get all the way down.
@copperfield42
@copperfield42 7 лет назад
this is 2 when x=sqrt(2): let Y= x^x^x^x^... -> Y=x^Y, take ln -> ln(Y)=Y*ln(x), now let Y be 2, then ln(2)=2*ln(x) ln(2)/2=ln(x) take the exponential and swap places x=e^(ln(2)/2) x=(e^ln(2))^(1/2) x=(2)^(1/2)=sqrt(2)
@ethandickson9490
@ethandickson9490 7 лет назад
That's cool, why specifically 2 though? This in general is x = (y)^(1/y) , y > 0, isn't it?
@copperfield42
@copperfield42 7 лет назад
because that what is what he ask us in the video... and as this is a infinite exponentiation there is a value of x to which this no longer a constant, I don't know exactly where this thing explode, but is obvious that it does... maybe for anything greater than sqrt(2)? but it sure does for x>=2
@cameodamaneo
@cameodamaneo 7 лет назад
I believe it blows up for x > e^(1/e) The function y = x^y Can be rewritten as x = y^(1/y) Of which the maximum is at x = e^(1/e), y = e He did found this maximum on his video about e^pi vs. pi^e.
@copperfield42
@copperfield42 7 лет назад
🤔 that sound very interesting, yeah, that must be it... other commentator said the same... PD: you mean x>e^(1/e) right?
@cameodamaneo
@cameodamaneo 7 лет назад
David Franco Whoops, yeah I did mean that haha. Thanks for pointing it out.
@link_z
@link_z 7 лет назад
Okay, I'll throw here my approach to the max. value where this function converges. First of all let's call y=x^x^x..... Therefore, y=x^y like he does in the video. After that, we take natural logarithms on both sides, so we get ln(y)=y*ln(x). Isolate ln(x) and we have ln(x)=ln(y)/y. Let's call f(y)=ln x = ln(y)/y and differentiate respect to y. f'(y)=(1-ln(y))/y^2 (after some simplification). The maximum of this function, is when f'(y)=0, (1-ln(y))/y^2=0 --> 1-ln(y)=0 --> y=e With the second derivative we can see it's a maximum and not a minimum. Coming back to ln(x)=ln(y)/y substitute y=e to obtain x=e^(1/e). Therefore x=e^(1/e) is the biggest value of x where the function y=x^x^x... converges, to y=e.
@link_z
@link_z 7 лет назад
If I've made any mistake please tell me!
@copperfield42
@copperfield42 7 лет назад
that sound about right, and as sqrt(2)
@dougr.2398
@dougr.2398 7 лет назад
The singular of maxima and minima are maximum and minimum.
@link_z
@link_z 7 лет назад
Doug R. Thanks! I didn't know
@SmileyMPV
@SmileyMPV 7 лет назад
Link_Z - Hardcore Indie Game Achievements You have only shown that x^x^x^... does not converge when x>e^(1/e), but you have not shown when it does converge. It turns out, for example that it does not converge for 0,05. To be exact, x^x^x^... converges exactly if 1/e^e≤x≤e^(1/e) or if x=-1.
@toasticide816
@toasticide816 7 лет назад
love your videos, theyre pretty much the main reason i love calculus. slight request if u havent already done so, integrate it (if its possible im not sure)
@semiawesomatic6064
@semiawesomatic6064 7 лет назад
Louis Moore you can't even integrate X^X and keep it in elementary functions. I doubt it can be done.
@toasticide816
@toasticide816 7 лет назад
semi awesomatic ah yes i remember now. thanks :) same goes for x^y then i assume
@semiawesomatic6064
@semiawesomatic6064 7 лет назад
Louis Moore unless it's a multivariable integration? Honestly don't know. But yeah in single variable it's impossible.
@tracyh5751
@tracyh5751 6 лет назад
If you integrate the derivative you will just obtain y again.
@JoseEduardo-rw2rh
@JoseEduardo-rw2rh 2 года назад
@@tracyh5751 + C
@Fematika
@Fematika 7 лет назад
You can find a formula for when it does converge. x^...^x = y_n has y_n = x^y_(n-1). In order for each of these to converge, it must be the case that y_(n+1) = y_n, meaning x^y_n = y_n. So, if it does converge, it'll converge to a solution of that. Now, y is actually is the Lambert-W function, the inverse function of xe^x, of -ln(x) over -x. This is because -W(-ln(x)) / ln(x) = y, and -y * ln(x) = W(-ln(x)), so x^(-y) = e^(W(-ln(x))), and thus x^(-y) * W(-ln(x)) = W(-ln(x)) * e^(W(-ln(x))) = -ln(x), so you get -W(-ln(x)) / ln(x) * x^(-y) = 1. or y = x^(y). You find that there are no solutions to the Lambert-W function outside of the interval [-1/e, e], See here: math.stackexchange.com/questions/1693561/for-what-values-does-this-method-converge-on-the-lambert-w-function. So, we must have -1 / e
@MuffinsAPlenty
@MuffinsAPlenty 7 лет назад
"So, we must have -1 / e
@Fematika
@Fematika 7 лет назад
Yep, you're right!
@sebastianmeier5002
@sebastianmeier5002 5 лет назад
What is "_"?
@paulgrimshaw6301
@paulgrimshaw6301 7 лет назад
I think y = x^y is only a valid function for 0< x ≤ 1 and x = e^(1/e). For 1 < x < e^(1/e) there are two possible values of y for each x, and for x > e^(1/e) y doesn't exist. You can see this by considering the inverse function, which is simply y = x^(1/x).
@tresslerdominick
@tresslerdominick 7 лет назад
Ya boi finna go impress some girls
@brndsaav10410
@brndsaav10410 7 лет назад
this feels like cheating...
@Nebukanezzer
@Nebukanezzer 5 лет назад
Why?
@cpotisch
@cpotisch 5 лет назад
@Pete Berg Yeah. Basically, all derivatives are cheating.
@pokrec
@pokrec 5 лет назад
All math looks like cheating. And it is the beauty of the math.
@blurb8397
@blurb8397 5 лет назад
It is, he didn’t prove the necessary assumptions to do it. I think this one requires uniform convergence, and this function series doesn’t even converge pointwise for most inputs in R. Edit: After further thought, maybe pointwise convergence is enough for this approach, I’m not sure though since I haven’t tried it on paper.
@HavaN5rus
@HavaN5rus 4 года назад
It's not even math. Firstly, he had do define what is x^x^x.... , because it's not a function from R to R. Then he had to define what is d/dx, and only after that he is allowed to do math
@gaurav.raj.mishra
@gaurav.raj.mishra 7 лет назад
it converges if x=1.
@mcmage5250
@mcmage5250 6 лет назад
Its 1.
@anjelpatel36
@anjelpatel36 6 лет назад
Then it would just be a constant function...
@TheLucasBurgel
@TheLucasBurgel 6 лет назад
converges for -1
@philb2972
@philb2972 6 лет назад
But what's the sign of y (or dy/dx, for that matter) for x
@mphayes98
@mphayes98 5 лет назад
Converges for 0
@VlanimationTales
@VlanimationTales 10 месяцев назад
You can also rewrite xy ln(x) at the end as x ln(x^y), which also equals x ln(y) because y = x^y. That would make the derivative in terms of x be [x^(x^(x^...))]^2 / [x - x ln(x^(x^(x^...)))].
@nestorv7627
@nestorv7627 7 лет назад
Cant wait to impress the guys with this ;)
@9wyn
@9wyn 7 лет назад
blackpenredpenbluepen
@Theraot
@Theraot 7 лет назад
I made that joke before... yet, reading it from you makes me want a to see a BlackPenRedPen & 3Blue1Brown collaboration
@Kassakohl
@Kassakohl 5 лет назад
@@Theraot BluePenBluePenBluePenPencil
@abramthiessen8749
@abramthiessen8749 5 лет назад
Well, it does converge for 0
@dolevgo8535
@dolevgo8535 7 лет назад
you could set x^x^x^x^... to be 2. then you could plug all the x^x^x^x which are not the base to be two. then you get x^2=2, or x=sqrt(2)
@rojastegulu
@rojastegulu 7 лет назад
Totally expecting this! Haha, I really love the humor of your channel and how enthusiastic are you in your videos, greetings from Colombia, keep it up :P
@jeromesnail
@jeromesnail 7 лет назад
Thank you
@SmileyMPV
@SmileyMPV 7 лет назад
jeromesnail You are actually wrong. For x=0,05 it will not converge for example.
@DoctorT144
@DoctorT144 4 года назад
It's actually only for { e^(-e) < x < e^(1/e) }
@quantum1861
@quantum1861 4 года назад
Convergent for all x such that 0
@KwongBaby
@KwongBaby 6 лет назад
thank you so much, you make integration/differentiation fun I love maths now
@bahouskouidri3164
@bahouskouidri3164 5 лет назад
You need to simplify more dy/dx=y²/x (1-lny) 😇😇😇great job
@mightychondriaofthecell3317
@mightychondriaofthecell3317 5 лет назад
🤣🤣🤣 I love that the motivation for this derivative is to impress girls. I'm definitely stealing that!
@bananewane1402
@bananewane1402 4 года назад
Only converges when X = 1. When X=1 dy/dx = 1 If X is 0 the bottom will be undefined due to the ln function. If X = -1 the bottom becomes imaginary (also due to ln function)
@AlexYoshidaVA
@AlexYoshidaVA 5 лет назад
Some say the x’s still go on to this day
@ahusky4498
@ahusky4498 5 лет назад
Can’t it still be simplified to dy/dx = ( x^(2 * x^x^x....) )/( x - ln(x^x^x...) ) If you bring the 2 into the top tower and you bring the “xy” part into the ln and then use exponent laws to get the tower again
@eugenecretu7257
@eugenecretu7257 Год назад
Really cool video, I wondered about this when I learned calculus last year. I wasn't to find as nice of a solution as you
@gillrowley7264
@gillrowley7264 3 года назад
Your videos are fascinating. Brings back my Calculus memories.
@Jacob-uy8ox
@Jacob-uy8ox 7 лет назад
4:35 hahah impressing girls be like, hey honey, look at this huge exponential function
@souhilaoughlis5832
@souhilaoughlis5832 3 года назад
3:38 that sign look like a big integral 😂
@wolfy7432
@wolfy7432 7 лет назад
No idea what all that means but you sounded smart. But in all seriousness, I am trying to self-teach myself calculus in hopes of getting a degree in physics. Although, it is very confusing hopefully, one day I will be able to understand all of your videos. Until then keep up the awesome videos ^-^
@blackpenredpen
@blackpenredpen 7 лет назад
Thank you! And I wish you the best of luck!
@misana77
@misana77 6 лет назад
Calculus isn't hard (espesially differentiation). I think the best way to self-teach calculus is watch whole calculus course somewhere like MIT OpenCourseWare or coursera.org Good luck!
@volcanic3104
@volcanic3104 6 лет назад
Im teaching myself with the AP calculus BC course on khanacademy.org The calc 1 topics are really cool and not difficult, Im gonna start infinite series soon If you want a more college-like course watch lectures and use the college course on khan Good luck!
@JonathanFetzerMagic
@JonathanFetzerMagic 5 лет назад
Watch '3 Blue 1 Brown' to get a good understanding of Calculus concepts, and 'Dr. Physics A' for the gist of many physics topics.
@TheRenegade...
@TheRenegade... 4 года назад
Do you understand it now?
@cameodamaneo
@cameodamaneo 7 лет назад
My hunch is that it converges for 0 < x < e^(1/e) 0 < y < e
@theginginator1488
@theginginator1488 7 лет назад
Cameron Pearce it’s actually the inverse relation of y=x^(1/x) Your domain is spot on, but you will actually get 2 values for 1
@cameodamaneo
@cameodamaneo 7 лет назад
TheGinginator14 I understand that. I was trying to paraphase because I am incredibly tired and I didn't think many would care.
@avivsilman3289
@avivsilman3289 7 лет назад
You get Lim(x→0+)(x^x^x^x...)=0 from that.
@cameodamaneo
@cameodamaneo 7 лет назад
Yes.
@ekadria-bo4962
@ekadria-bo4962 7 лет назад
The domain of x is e^-1 < x < e^e^-1
@piguyalamode164
@piguyalamode164 7 лет назад
x^x^x^x... converges when x < e^(1/e) because if you set x^x^x^x^x...= b then x = b^(1/b) and the maximum value of b^(1/b) is b=e.
@justafanofalphabetlore
@justafanofalphabetlore Год назад
Including derivate of this infinite tetration is also calculate the main function -W(-ln(x))/(ln(x))
@taruntomar7795
@taruntomar7795 7 лет назад
that is just awesome no wait a minute its better than that can't wait for next video
@akshaygavini2031
@akshaygavini2031 7 лет назад
Zero dislikes. You've somehow hacked RU-vid.
@blackpenredpen
@blackpenredpen 7 лет назад
Bc of Oreo!
@yourstandardgoogleaccount303
@yourstandardgoogleaccount303 6 лет назад
Who doesn't like bunnies? :p
@GonzaLiitoo2012
@GonzaLiitoo2012 5 лет назад
71
@326inminecraft
@326inminecraft 5 лет назад
Akshay Gavini shouldn’t have said it
@ThichMauXanh
@ThichMauXanh 4 года назад
just added one dislike for fun
@mrKreuzfeld
@mrKreuzfeld 6 лет назад
I would love some comments about which functionspace this function belongs to. It is not clear to me that it is differentiable. It looks to me like it is only defined between 0
@dustinbachstein3729
@dustinbachstein3729 3 года назад
Fun fact: x^x^x^... can be expressed by solving y=x^y for y, using Lambert's W function! Result: x^x^...=e^(-W(-ln(x))) for x in (0, e^(1/e)]
@Whysicist
@Whysicist 3 года назад
Infinite Exponentials converge in, [0.06,1.4466], Barrows ~1936. His notation is, k = a ^ k, for , a, in, [0.06,1.4466], as i remember it…
@hewhomustnotbenamed5912
@hewhomustnotbenamed5912 4 года назад
I cam up with y^2/(y^(1/y)*(1-ln(y)) I did it by drawing a line tangent to y=x^y (since that is equivalent to the derivative) and then making a right hand triangle out of it with dx and dy as the legs. From their I just imagined dx/dy(x=y^(1/y)). It turned out to be equivalent to 1/tan(z) where is the angle formed by the dx leg going through the tangent line. From their I just worked out tan(z) so that I get dy/dx. That was equivalent to just taking the reciprocal of dx/dy(x=y^(1/y)). And that gave me dy/dx(y=x^y)=1/(1/y^2*y^(1/y)*(1-ln(y))) I feel skeptical of my answer but it checked out with the 2 tests I did. Can anyone confirm that my trigonometry method works?
@earthbjornnahkaimurrao9542
@earthbjornnahkaimurrao9542 6 лет назад
I plotted the first 71 tetrations of x, really quite interesting. Also, the first time I tried I made the mistake of iterating (((x^x)^x)^x ..... Is there a term for this? The limit is equal to 1 for 0
@senseibuh527
@senseibuh527 5 лет назад
That equals x ^ (x ^infinity)
@StarEclipse506
@StarEclipse506 6 лет назад
"I have no idea what this is anymore" Your videos are awesome!!
@bailey125
@bailey125 5 лет назад
Say you have an even number of infinite exponentials, if you plug in x = 0, then y = 1, however if there are an odd number of exponentials than if you plug in x = 0, then y = 0. It's like saying whats the answer to Grandi's series if you start with 0 as he first term.
@billtaylor4
@billtaylor4 3 года назад
Me: I can take the Derivative of infinite power tower x^x^x^... Girl: That's cool. Let's integrate together, baby!
@mikeburns6603
@mikeburns6603 4 года назад
The function y only exists on the interval (0,1]. And it appears to always be equal to 1. Therefore it's a constant and the derivative is zero. Note: pay attention to the domain of the function.
@mikeburns6603
@mikeburns6603 4 года назад
On closer inspection, it only seems to be defined for x=1. Since the function only exists at one point, it has no derivative.
@MatesMike
@MatesMike 4 года назад
And which is the convergence domain of this function? Isnt it identically equal to 1?
@RileyGallagher-ce4rq
@RileyGallagher-ce4rq 5 месяцев назад
You can also write the final answer as y / x (1 - ln(y)), since y ln(x) = ln(xʸ) = ln(y).
@modolief
@modolief 5 лет назад
This was very entertaining, thanks!
@Mansch007
@Mansch007 5 лет назад
the derivative of y(x)=(x^x^x^x^...)*(lnx) seems much nicer and handier
@blackpenredpen
@blackpenredpen 5 лет назад
: )
@soufian2733
@soufian2733 6 лет назад
Weird. In the denominator of the final answer, if you bring the factors of ln(x) inside as its exponents, won't you just end up with ln(y) ? I feel like there's an infinite amount of answers
@christopherglanville1619
@christopherglanville1619 4 года назад
"if you want to make this slightly more interesting so you can impress girls" honestly one of the funniest things I've ever heard.
@_radix_
@_radix_ 4 года назад
Man, you're just amazing and your content is both enjoyable and useful at the same time
@wehitextracellularidiombit4907
Not so useful for me but enjoyable yes 🙂
@MisutaaPurinsu
@MisutaaPurinsu 4 года назад
I have a question. I haven't studied that kind of derivative yet so I came up with this: f'(x)=((f^(-1))')^-1 (x) The inverse of f(x)=x^x^x^x^x^... -> y=x^y -> x=y^x; f^-1(x)=y=e^(lnx /x) y=f^-1 ' (x)=e^(lnx /x) * (1-lnx)/x^2 f'=((f^-1)' )^-1-> x=e^(lny /y) * (1-ln y)/y^2 How horrendous is that answer? (It actually works if you want to find the tangend line on y=a)
@Paul-222
@Paul-222 Год назад
Why not factor out the common factor of x in the denominator and then make all those red x’s the power of the x that is the argument of the ln function. Then the denominator would be x(1 - ln [infinite tetration of x]). You could also move that 2 in your numerator inside the parentheses and subtract 1 by factoring the x out of the denominator. The numerator would be x to [(twice the infinite tetration of x) - 1)]
@karinano1stan
@karinano1stan 4 года назад
The more formal form of result is W(-ln(x))^2/(x ln^2(x) (W(-ln(x)) + 1)). W(x) is product log function, also called Lambert function.
@SlipperyTeeth
@SlipperyTeeth 5 лет назад
y^2=x^(2y) x*y=x^(y+1)
@batman1155
@batman1155 4 года назад
You forgot to check if x-xyln(x)=0 because if it is, your have to exclude values of x and y.
@morbidmanatee5550
@morbidmanatee5550 7 лет назад
y=x^x^x^... is the same as y=x^y x=y^(1/y) When y=2, x=sqrt(2) Try different values of x :) Has very interesting properties.
@mattheworchard481
@mattheworchard481 2 года назад
Aka derivative of x tetrated to infinity d/dx( ᪲x) Also, ᪲x is equal to the inverse of ˣ√x
@birupakhyaroychowdhury974
@birupakhyaroychowdhury974 5 лет назад
I loved Oreo much more than ur differentiation...
@saumitup6524
@saumitup6524 5 лет назад
I'd say they're both equally (Def.) Lovely
@TheGundeck
@TheGundeck 5 лет назад
It converges for values of x between 0 and about 1.445
@charlesbromberick4247
@charlesbromberick4247 3 года назад
Rabbits in the wild generally live about one year; domesticated rabbits commonly live up to ten years.
@aznvi3tprid3
@aznvi3tprid3 2 года назад
May you do a video about the nth term of the sequence {-1/3,1/3,0,-1/3,1/3,0….}
@lars9168
@lars9168 Год назад
i started studying mechanical engineering but since we also have math classes I kinda regret not simply going for pure MATH :D
@fichte5061
@fichte5061 4 года назад
4:54 you said 'blue' so seriously xD
@kyamb3890
@kyamb3890 Год назад
Instead of implicit differentiation I just move all to one side and do -partial x/ partial y = dy/dx
@nathanaelmccooeye3204
@nathanaelmccooeye3204 5 лет назад
Hi, how would you plug in and solve for a value of x? I guess go into the definitions of infinities? I would love to see videos on that! :D
@General12th
@General12th 7 лет назад
Could we use implicit differentiation using Newton's notation? Because it seems like Leibniz's notation is way more useful.
@ValkyRiver
@ValkyRiver Год назад
x converges between 1/(e^e) and the eth root of e.
@patuchitogomez6790
@patuchitogomez6790 6 лет назад
You're very brillant for math You won my like
@blackpenredpen
@blackpenredpen 6 лет назад
Thanks!
@patuchitogomez6790
@patuchitogomez6790 6 лет назад
blackpenredpen You're welcome
@timepaintertunebird8160
@timepaintertunebird8160 4 года назад
Pretty sure the range -1 to 1 converges but idk if anything else does.
@odbhut424
@odbhut424 6 лет назад
For 0
@rafaeldubois8192
@rafaeldubois8192 7 лет назад
If x^x^x^x^...=2 Then x²=2 So x=√2 √2^√2^√2^...=2 If you plug that to the calculator, you have to first write √2, and then do √2^ANS a lot of times, and it's true, it converges :) Interestingly enough, tough... x^x^x^...=4 x⁴=4 x=⁴√4=4^¼=(2²)^¼=2^½ x=√2 Can someone explain this paradox? Could it have something to do with the ±?
@link_z
@link_z 7 лет назад
it converges only for x in [e^-e , e^(1/e)]. Outside this range it diverges
@leoitshere
@leoitshere 7 лет назад
There is no paradox. Remember that what you actually have is the expression y = x^y. Now notice that 2 = sqrt(2)^2 but also 4 = sqrt(2)^4. There is no paradox, that is just how numbers work.
@rafaeldubois8192
@rafaeldubois8192 7 лет назад
Link_Z - Hardcore Indie Game Achievements Wow, I didn't know that. How do you prove it?
@link_z
@link_z 7 лет назад
I'm going to wait and see if blackpenredpen wants to make a video about that. If he doesn't in a few days I'll prove it in a comment for you :)
@leoitshere
@leoitshere 7 лет назад
Well I will add that there is a "paradox". The thing is that for some x (in your case sqrt(2)) the equation y = x^y has multiple solutions. You can see that y=4 and y=2 are both solutions when x= sqrt(2). But that is algebraic. When you see y as the limit of the sequence x,x^x,x^x^x,x^x^x^x... then that sequence either converges or not. And if it converges it can be proven that the limit must be unique. Therefore, if x^x^x^x... converges then it will only be able to reach one of the solutions to y = x^y, not all of them. You haven't really "proven" that sqrt(2)^sqrt(2)^sqrt(2) = 4. What you actually did was start with x^x^x^x... = 4 but then you replaced that to x^4 = 4, so now you are not doing a sequence, you are solving the algebraic equation y= x^y.
@yakov9ify
@yakov9ify 6 лет назад
I found something curious. if you graph y=x^y you can see that y->0 as x->0, while if you graph x^x^x.... with a finite amount of Xs when x->0, y->0 if you have an odd number of total Xs and y->1 if you have an even number of total Xs. This seems to imply that infinity is odd since it behaves like an odd number. obviously infinity isnt odd or even so why does it aact like an odd number?
@srpenguinbr
@srpenguinbr 5 лет назад
You missed the opportunity to define y of x using Lambert W function.
@userBBB
@userBBB 4 года назад
4:34 geek to the power of infinity
@davidwright8432
@davidwright8432 7 лет назад
Lots of fun and completely AWESOME! I'd love to show Gottfried Leibniz this. He could taunt Newton with it!
@GottfriedLeibnizYT
@GottfriedLeibnizYT 5 лет назад
calculate the limit of this derivative as x approaches 0
@kalebbruwer
@kalebbruwer 6 лет назад
If you graph this, you will have a flat line between 0 and 1(with those points excluded), It will jump to 1 at 1 and then be gone, because how do you differentiate infinities? For 0 it won't exist. Between -1 and 0 the original function converges to 1, so you will have a flat line at 0 again. Left of -1, you have a diverging, oscillating mess. Good luck differentiating that.
@ori1618
@ori1618 4 года назад
I believe it converges for any value between one over e and the e root of e
@pfeffer1729
@pfeffer1729 7 лет назад
It's basically 0
@Adem-zb3uo
@Adem-zb3uo 5 лет назад
Bro this guy is straight up hacking math
@Supware
@Supware 6 лет назад
When x is very large, (x^x^x^...)^2 will also be very large and positive, while x-x(x^x^x^...)ln x will be very large and negative, implying that dy/dx itself is negative (I guess for all x > 1?). That feels really counter-intuitive to me; the curve looks like it should be growing at a vast rate. Could you explain what's going on there? Also, is dy/dx defined in the reals for 0 < x < 1? Is there a clever way to find y(1/2) or y'(1/2) for example? Thankyou for the interesting video :)
@trangium
@trangium 5 лет назад
Because the function does not converge for very large values of x. Any x above 1.445... will diverge
@ryotanada
@ryotanada 4 года назад
I got this as one of my practise questions when I was preparing for my uni entrance exams in a course...
@999Aadil-Op
@999Aadil-Op 5 лет назад
What's that gadget in your hand ...is it Telepathy?
@theusa4052
@theusa4052 4 года назад
I’m in Geometry. I am not sure why I am listening to Calculus stuff, but I can what is going on a little bit. Comment section: Should I stop? If I should not, please explain why this is a good idea to continue watching this.
@paulchapman8023
@paulchapman8023 4 года назад
If you enjoy watching this sort of thing, that’s reason enough to keep watching this sort of thing.
@RAG981
@RAG981 7 лет назад
The "function" does not converge for x1, so the function's domain must be restricted. Thanks for the workout, I love your flights of fancy.
@michielhorikx9863
@michielhorikx9863 7 лет назад
Ken Garner I tried it and I think it does converge, but really slowly. I got x=0.067 to converge (reasonably) when the stack is about 2000 x's high. I think it converges for the half-open interval (0, e^(1/e)].
@angelmendez-rivera351
@angelmendez-rivera351 7 лет назад
It has been proven it converges for (e^[-e], e^[1/e])
@michielhorikx9863
@michielhorikx9863 7 лет назад
Angel Mendez-Rivera Hm that seems right. I couldn't get 0.65 to converge really, which is just outside the range. Do you mean the open or closed interval?
@angelmendez-rivera351
@angelmendez-rivera351 7 лет назад
Michiel Horikx It is an open interval. Although that’s irrelevant, since regardless of the interval, no number outside of it should yield convergence. Not sure why would you test it with a number outside.
@michielhorikx9863
@michielhorikx9863 7 лет назад
Angel Mendez-Rivera Open and closed refer to the endpoints, so in other words, are e^-e and e^(1/e) included or not? The second one works as far as I know.
@deeptochatterjee532
@deeptochatterjee532 6 лет назад
Girls don't feel restricted by what he said, feel free to impress me with this (seriously I love math so much)
@davidwhitecross1021
@davidwhitecross1021 6 лет назад
this is gold you're great!
@telotawa
@telotawa 4 года назад
at the end: y*ln(x)=ln(y) tho, it looks nicer that way especially when you do it as (x^x^x...)^2 / (x * (1 - ln(x^x^x...))
@timewilltell2225
@timewilltell2225 6 лет назад
5:08 i have no idea wat is dis :D
@yahyaguzide
@yahyaguzide 7 лет назад
BlackpenRedpen can you integrate this ??!
Далее
derivative of tetration of x (hyperpower)
10:58
Просмотров 523 тыс.
World’s Tallest Man VS Shortest Woman!
15:07
Просмотров 16 млн
Differentiate x^x^x^x
16:28
Просмотров 39 тыс.
e^pi vs pi^e (no calculator)
10:59
Просмотров 748 тыс.
We must do this do this carefully!
11:59
Просмотров 303 тыс.
Derivative of  The Factorial Function
6:47
Просмотров 104 тыс.
The Bernoulli Integral is ridiculous
10:00
Просмотров 703 тыс.
Line Integrals Are Simpler Than You Think
21:02
Просмотров 46 тыс.
How Cauchy would find the maximum of sqrt(x)+sqrt(y)
9:45